LSAT and Law School Admissions Forum

Get expert LSAT preparation and law school admissions advice from PowerScore Test Preparation.

 Nikki Siclunov
PowerScore Staff
  • PowerScore Staff
  • Posts: 1362
  • Joined: Aug 02, 2011
|
#14801
Hi nattycat,

Thanks for your question. I can see that you misunderstood the conclusion of the argument. The conclusion states,
It is best not to take a strong position on an issue unless you have considered all important evidence conflicting with that position.
Applying the Unless Equation, the statement modified by the word "unless" should be understood as the necessary condition. The remainder must be negated to become the sufficient condition:
Take strong position :arrow: Consider all important evidence
In other words, to take a strong position on an issue, you must consider all important evidence conflicting with that position.

Your interpretation of the conclusion is the Mistaken Negation of this idea, which is why you thought answer choice (A) would be attractive (that would be the Mistaken Reversal). Note that the Mistaken Negation and the Mistaken Reversal are contrapositives of each other.

Let me know if this clears things up!
 hdavidson
  • Posts: 2
  • Joined: Jan 31, 2015
|
#18135
I am having a difficult time with #18 in the LR1 section of Preptest 54 (June 2008)

I selected A and crossed out B,D and E. The correct answer was C yet I cannot see why this answer choice is superior to A. Could somebody please help me understand this?
 David Boyle
PowerScore Staff
  • PowerScore Staff
  • Posts: 836
  • Joined: Jun 07, 2013
|
#18137
hdavidson wrote:I am having a difficult time with #18 in the LR1 section of Preptest 54 (June 2008)

I selected A and crossed out B,D and E. The correct answer was C yet I cannot see why this answer choice is superior to A. Could somebody please help me understand this?
Hello hdavidson,

Answer A may be a mistaken reversal, saying, essentially, that if you understand something, it's o.k. to take a strong position on it. But answer C says the opposite, that taking a strong position requires (needs) understanding. (I.e., that's the contrapositive of answer C, "Anyone who does not understand an issue fully should avoid taking a strong position on it.")
After all, the stimulus says, "Thus, it is best not to take a strong position on an issue unless one has already considered all important evidence conflicting with that position.", diagrammable as "take strong position :arrow: considered evidence". This is closest to answer choice C of any of the answer choices, especially considering "But in order to understand an issue fully, it is essential to consider such evidence impartially. ", diagrammable as "understand :arrow: consider evidence". So if you didn't consider the evidence, how could you understand the issue fully?

Hope this helps,
David
 HappySaban_RTR
  • Posts: 5
  • Joined: Nov 12, 2014
|
#19890
also had issues with this question at first. I understand the diagramming and the assumption we need to connect the two premises. However how do we determine when to treat a principle question like an assumption question? Is there any indicator in the stimulus or question stem I am missing? I feel like this is not treated as a typical principle question at all. I would appreciate anybody's input. thanks guys.
 Jon Denning
PowerScore Staff
  • PowerScore Staff
  • Posts: 904
  • Joined: Apr 11, 2011
|
#19934
Hey happy,

Thanks for the question! I've enjoyed following along with this conversation, but couldn't fight the urge to jump in participate :)

The language of the question stem here is pretty typical for a Strengthen or Justify Principle question, where we are told that the specific information in the stimulus most closely conforms to a principle in the answer choices. That means you'll be using a broad rule presented in an answer choice to support or prove the reasoning above.

In Justify questions, as Nikki notes earlier in this thread, that often means connecting information in the premises to new information in the conclusion, allowing that "new" info to be reasonably supported and thus proven as true. And that's exactly what happens here!

So always be on the lookout for a "gap" whenever you see argumentation—new information in a conclusion is trouble for an argument, and needs to be tethered to something in a premise to have a chance at being acceptable—but particularly in Assumption and Justify, as filling gaps is one of the most common tasks in those two question types.

I hope that helps!
 Sherry001
  • Posts: 81
  • Joined: Aug 18, 2014
|
#20213
Hello;
I found this principle question very difficult to answer . And it really felt like justify question to me ! I think I struggled because I didn't know how to properly connect the conditional chains :( and I only chose c because most of the other answers had the word "reasonable ".
Could you please look at my conditional chains ?

P1- taking a strong position on an issue makes it likely that one would misinterpret.
P2- to understand --> consider evidence

C: strong position -->consider all evidence


Linking the chains gave me these : (but which one is the right one )?

Understand -> strong position -> all evidence
OR
ST -> understand --> all evidence


a) reasonable is not discussed
B)we only one should avoid taking strong positions
C) contrapositive of our chain .
D)what about avoiding strong position ? This doesn't get at that.
E)reasonable is not discussed .



Thanks so much
Sherry
 Anthony Esposito
PowerScore Staff
  • PowerScore Staff
  • Posts: 14
  • Joined: Sep 16, 2015
|
#20214
Hi Sherry,

Let me see if I can help you out with this principle question. Just to ensure we've got the fundamentals, a principle is a broad rule that specifies what actions or judgments are correct in certain situations. Also, remember, this is a Must Be True question with a principle "overlay" as principle questions are not a separate question type, but more like an "add-on" like conditionality or causality.

Since, at its heart, this is a Must Be True question, the correct answer choice must pass The Fact Test. We also know that the correct answer choice is the answer choice that presents a principle that the stimulus must adhere to. The correct answer choice is answer choice (C). Let's see why . . .

I would attack this problem without worrying too much about combining conditional chains. (Side note: You did a great job identifying the two premises, the conclusion, and the often-missed Unless Equation in the conclusion. :) )

According to the conclusion in the stimulus (the last sentence, note conclusion indicator "thus"), one should not take a strong position on an issue unless one has already considered all important evidence conflicting with that position. Why do we need to consider all important conflicting evidence first? Because of what we learn in sentence #1: Taking a strong position is likely to make us miss conflicting evidence. If we have considered all the evidence fairly (sentence #2), then we need to consider conflicting evidence impartially.

The reasoning in the stimulus follows the rule from answer choice (C): Anyone who does not understand an issue fully should avoid taking a strong position on it.

Hope this alternative approach to this tricky Must Be True-Principle question helps!

Anthony
PowerScore Instructor
Athens, GA
 tld5061
  • Posts: 11
  • Joined: Apr 06, 2017
|
#38037
Hi,

I understand how to diagram the statements but am wondering how you know that the correct order of the connecting chain is: strong position > understand fully. If you are connecting two sufficient conditions, does the sufficient condition of the conclusion always come first in the connecting chain? I understand when I connect two necessary conditions the necessary condition in the premise always come first so just trying to make sure I'm thinking about this correctly (ie: the end points of the conclusion should always remain the endpoints of the chain).

Thanks!
 Francis O'Rourke
PowerScore Staff
  • PowerScore Staff
  • Posts: 471
  • Joined: Mar 10, 2017
|
#38113
If you don't know what order to link these terms up, it will not take many tries to figure it out through trial and error.

Since we are looking to justify the conclusion with a principle, we need to connect ideas from the premise with ideas from the conclusion. Usually this will mean looking for ideas that were brought up only in the premises and connecting them with ideas that were brought up only in the conclusion. This question can be solved in exactly this way.

First pick out that the premise and conclusion share the term "consider all evidence," and then link the other ideas that were brought up only in the premise or only in the conclusion: "understand fully" and "take a strong position." There are two ways to connect these statements using conditional reasoning.
  1. Understand fully :arrow: Take a strong position
  2. Take a strong position :arrow: Understand fully

First consider what the first statement does to the argument. With this statement, we would know something that would have to occur if you "understand something fully." However we wouldn't have any additional information about what would have to occur if you "take a strong position." Since the conclusion is about what must happen if you take a strong position, then this conditional does not justify the conclusion.

The second statement above does provide additional information about what must occur if you take a strong position. Namely it tells us that you must understand the issue fully. We should then immediately chain together this statement with the conditional statement from the premise to find that this statement tells us that if you take a strong position, then you must consider all information.

I would be careful about generalizing too much about justify questions. However, in questions like this, in which we need to connect two sufficient conditions, in order to say that the necessary condition of the former is also necessary for the latter, then you will most likely order the information in this way. This will get more complicated when you have more than two conditional statements to deal with, so don't rely too heavily on memorizing patterns like this.
 jessicamorehead
  • Posts: 84
  • Joined: Jul 07, 2017
|
#38957
Quick question: How do you know which way the assumption goes?

P: Understand --> Consider Evidence
C: Strong Position --> Consider Evidence

I see that we have to link up "Understand" and "Strong position," but how do you know which is sufficient and which is necessary? When I think about it logically, I understand that IF you want to take a strong position, then it is NECESSARY you understand. So I can get to the assumption that way, but could I also always assume that the conclusion is the sufficient condition and the premise is the necessary one??? I feel like that would be a quick hack to get to the same thing.

Assumption: Strong position --> Understand

Get the most out of your LSAT Prep Plus subscription.

Analyze and track your performance with our Testing and Analytics Package.